An experiment observing the growth of two germ strand populations finds these patterns.

• The population of the first germ is represented by the function A(x) = (1.3)^x+9

• The population of the second germ is represented by the function B (x) = ^4x+1

Which function best represents function R, the ratio of the population of the second germ to the population of the first germ?

A. R(x) = (1.3)^3x-8
B
An experiment observing the growth of two germ strand populations finds these patterns.

• The population of the first germ is represented by the function A(x) = (1.3)^x+9

• The population of the second germ is represented by the function B (x) = ^4x+1

Which function best represents function R, the ratio of the population of the second germ to the population of the first germ?

A. R(x) = (1.3)^3x-8
B. R(x) = (1.3)^5x+10
C.R(x) = (2.6) ^3x-8
D. R(X) =(2.6) ^4x^2+9​

An Experiment Observing The Growth Of Two Germ Strand Populations Finds These Patterns. The Population

Answers

Answer 1

The right answer, based on the provided assertion, is (C) R(x) = 2.6³ˣ⁻⁸.

What does an arithmetic the expression mean?

Mathematical expressions consist of at least two integers or factors, a minimum of one math procedure, and a sentence. It's possible to multiply, divide, add, or remove with this mathematical procedure. An expression's form is as follows: Expression: (Math Operator, Number/Variable, Math Operator)

To find the ratio of the population of the second germ to the population of the first germ, we can divide the function for the population of the second germ by the function for the population of the first germ. That is:

R(x) = B(x) / A(x)

Plugging in the given functions, we get:

R(x) = (x+1) / (1.3)ˣ⁺⁹

To simplify this expression, we can use the properties of exponents and rewrite it as:

R(x) = 2.6ˣ⁻⁸

Therefore, the function that best represents the ratio of the population of the second germ to the population of the first germ is R(x) = 2.6ˣ⁻⁸

So, the correct option is (C) R(x) = (2.6)³ˣ⁻⁸.

To know more about Expression visit:

https://brainly.com/question/1859113

#SPJ1


Related Questions

In the following system, what is the x-value of the solution?

y = 4

y = 2x - 5

Answers

Answer:

x = 9/2

Step-by-step explanation:

As both equations are set equal to "y", we can set the equations equal to each other and solve for the x-value:

[tex]4=2x-5\\9=2x\\x=\frac{9}{2}[/tex]

what’s the volume of each rectangular prism
1. 4 yd 7 yd 2 yd

Answers

Answer: volume would be 56

Step-by-step explanation:

Volume = length X width X height

The answer would be 19.6 because the equation would be 1.4x7x2=19.6

Tamara is painting a rectangular picture that is 6 inches wide by 4 3/4 inches long. What is the area of the picture

Answers

Now we can find the area:

Area = length x width

Area = (19/4) x 6

Area = 114/4

Area = 28.5

Therefore, the area of the picture is 28.5 square inches.

What does the mathematical concept of "area" mean?

The area of a planar figure is the region that its perimeter includes. The quantity of unit squares that completely encircle the surface of a closed figure is its area. Square units like cm2 and m2 are used to measure area.

The area of a rectangle is found by multiplying its length by its width. In this case, the width is 6 inches and the length is 4 3/4 inches. We can convert the length to an improper fraction to make it easier to multiply:

4 3/4 = 19/4

Now we can find the area:

Area = length x width

Area = (19/4) x 6

Area = 114/4

Area = 28.5

Therefore, the area of the picture is 28.5 square inches.

To know more about area visit:

https://brainly.com/question/27683633

#SPJ1

The area of the rectangular picture painted by Tamara is 28.5 in.²

What is a rectangle?

The internal angles of a rectangle, which has four sides, are all precisely 90 degrees. The two sides come together at a straight angle at each corner or vertex. The rectangle differs from a square because its two opposite sides are of equal length. Rectangles are flat, two-dimensional shapes. The diagonals of the rectangle's symmetrical shape are of equal length. The rectangle will be split into two right-angle triangles by a diagonal.

The region covered by a two-dimensional plane is known as its area. It has a square unit of measurement. As a result, the rectangle's area is the space enclosed by its exterior lines. It is the same as the length times the width calculation.

Given,

  The width of the rectangular picture = 6 inches

The length of the rectangular picture = 4 3/4 inches = 19/4 inches

The area of the picture = Area of the rectangle = Length * breadth

                                                                      = 19/4 * 6 = 57/2 = 28.5 in.²

Therefore the area of the rectangular picture painted by Tamara is

28.5 in.²

To learn more about rectangles, follow the link.

https://brainly.com/question/25292087

#SPJ1

The probability of guessing right on a 4-option multiple choice item is 1/4 or 0.25. If a quiz has 10 questions. Determine the mean = (exact answer, no rounding) variance = (exact answer, no rounding) standard deviation = und to one decimal place) imal place)

Answers

The probability of getting more than 5 questions right in the quiz is

0.00011445.

Probability

Probability is simply how likely something is to happen. Whenever we're unsure about the outcome of an event, we can talk about the probabilities of certain outcomes—how likely they are.

the probability of guessing right on a 4-option multiple choice item is= 1/4

= 0.25

A quiz contains 10 questions.

now, the probability of getting 5 right answers as below,

Required Probability = P(X = 6) + P(X = 7) + P(X = 8) + P(X = 9) + P(X = 10)

so, this is the probability to come right answers,

Required Probability  = (1/4)^6 (3/4)^4 + (1/4)^7 (3/4)^3 + (1/4)^8 (3/4)^2 + (1/4)^9 (3/4)^1 + (1/4)^10

Required Probability = 0.000077 + 0.000025 + 0.0000086 + 0.0000029 + 0.00000095

after adding all values we get the required probability,

Required Probability = 0.00011445.

Learn more about probability here :-

https://brainly.com/question/11234923

#SPJ4

the complete and correct question is ,

the probability of guessing right on a 4-option multiple choice item is 1/4 or 0.25. a quiz contains 10 questions. find the probability of getting more than 5 right.

a distribution table for the scores on an exam is shown below. the second row says that 10% of the students scored between 60 and 70. fill in the blanks in the height column. do not include units.

Answers

The median for the distribution table is falls within the range of 60-70 points.

Understanding the concept of median is crucial in analyzing data and making statistical inferences.

From the given distribution table, we can see that the percentage of students scoring between 0-60 points is 15%, between 60-70 points is 10%, between 70-80 points is 25%, between 80-90 points is 30%, and between 90-100 points is 20%. To calculate the cumulative percentage, we can add up the percentages from left to right.

For instance, the cumulative percentage of students scoring between 0-70 points is

=> 15% + 10% = 25%.

Similarly, the cumulative percentage of students scoring between 0-80 points is

=> 15% + 10% + 25% = 50%.

We can repeat this process to find the cumulative percentage for each score range.

Once we have the cumulative percentages, we can find the median score. In this case, since the total percentage is 100%, the median score would be the point at which the cumulative percentage reaches 50%. We can use the height column to calculate this value.

Starting from the lowest score range, we can add the heights until the cumulative percentage exceeds 50%. The point where this happens is the median score.

Moving on to the next score range, the cumulative percentage for scores between 0-70 points is 25%, which is greater than 50%. Therefore, the median score falls within the range of 60-70 points.

To know more about distribution here.

https://brainly.com/question/29062095

#SPJ4

Complete Question:

A distribution table for the scores on an exam is shown below. The second row says that 10% of the students scored between 60 and 70. Fill in the blanks in the height column. Do not include units.

Points (Width) % 0-60  60-70 70-80 80-90 90-100

(Area) Height=Area/Width (% per point)  15  10  25 30  20

What is the median score?

A bag contains 6 black marbles, 14 white marbles, and 5 grey marbles.
You pick one without looking. What is the probability that the marble will be
either white OR black? Answer as a fraction or whole number in lowest
form.

Answers

4/5 chance that the marble you pick will be either white or black

6 + 14 + 5 = 25

Now you take 25 and put 20 over it to get the fraction of 20/25.

Simplify this answer by dividing both of these numbers by 5 to get the fraction 4/5

find the conditional expectation of alpha x poisson and gamma distribution law of iterated expectation

Answers

The conditional expectation of αX given Y=y is (αλy / β + αλ)^α * e^(-(β+αλ)/βy) and the Law of Iterated Expectation gives E[αX] = (αλ / β + αλ)^α * (β / (β+αλ))^α * (Γ(α) / β).

Let X be a Poisson random variable with parameter λ and Y be a Gamma random variable with parameters α and β. We want to find the conditional expectation of αX given Y=y and then apply the Law of Iterated Expectation to find E[αX].

The conditional probability density function of αX given Y=y is:

fX|Y(x|y) = P(X=x|Y=y) = P(X=x,Y=y) / P(Y=y)

Since X and Y are independent, we have:

P(X=x,Y=y) = P(X=x)P(Y=y)

Therefore, the conditional probability density function of αX given Y=y is:

fX|Y(x|y) = (λ^x/x!) * (β^α / Γ(α)) * y^(α-1) * e^(-βy) / (λ^y * e^(-λ) * β^α / Γ(α))

fX|Y(x|y) = (λ^x / x!) * (y^α * e^(-βy)) / (Γ(α) * λ^y * β^α)

Now we can calculate the conditional expectation of αX given Y=y:

E[αX|Y=y] = ∑ x=0^∞ αx * fX|Y(x|y)

E[αX|Y=y] = ∑ x=0^∞ αx * (λ^x / x!) * (y^α * e^(-βy)) / (Γ(α) * λ^y * β^α)

E[αX|Y=y] = (αλy / β + αλ)^α * e^(-(β+αλ)/βy)

Now we can apply the Law of Iterated Expectation to find E[αX]:

E[αX] = E[E[αX|Y]]

E[αX] = E[(αλY / β + αλ)^α * e^(-(β+αλ)/βY)]

Since Y is a continuous random variable, we need to integrate over all possible values of Y:

E[αX] = ∫ 0^∞ (αλy / β + αλ)^α * e^(-(β+αλ)/βy) * (β^α / Γ(α)) * y^(α-1) * e^(-βy) dy

E[αX] = (αλ / β + αλ)^α * (β / (β+αλ))^α * (Γ(α) / β)

For such more question on expectation

https://brainly.com/question/24305645

#SPJ4

a chef used 1/4 cup of milk for one recipe. Then she used 2 cups of milk for 5 recipes. The total number of cups of milk the chef used can be found by expression.
1/4+(2x5)

Answers

Answer:

Step-by-step explanation:

Answer:

The chef used a total of 10.25 cups of milk.

Step-by-step explanation:

By adding together the specific amounts the chef used in each recipe, one can determine the total number of cups of milk she consumed. In order to achieve this, we can condense the phrase:

1/4 + (2 x 5) = 1/4 + 10

The two amounts on the right side of the equation can then be added:

1/4 + 10 = 10.25

To learn more regarding this question visit: https://brainly.com/question/19162363

How do we prove by contrapositive that if n is an integer and n^3+5 is odd, then n is even?

Answers

To prove by contrapositive, we need to show that if n is odd, then n^3+5 is even.

The contrapositive of the original statement is logically equivalent to the original statement.  if we can prove the contrapositive, then we have also proven the original statement.

Proof by contrapositive:

Assume that n is an odd integer, which means that n can be written in the form n = 2k + 1, where k is an integer.

Then, n^3+5 = (2k+1)^3 + 5

= 8k^3 + 12k^2 + 6k + 1 + 5

= 8k^3 + 12k^2 + 6k + 6

We can factor out a 2 from the last two terms of this expression:

n³ + 5 = 2(4k³ + 6k²)

Since 4k³ + 6k² + 3k + 3 is an integer, we can see that n³ + 5 is an even integer, which means that n is even by definition.

we have proven that if n is an integer and n³ + 5 is odd, then n is even by contrapositive.

n^3+5 = 2(4k^3 + 6k^2 + 3k + 3)

Since 4k^3 + 6k^2 + 3k + 3 is an integer, n^3+5 is even.

we have shown that if n is odd, then n^3+5 is even, which is the contrapositive of the statement we were asked to prove.

To learn more about contrapositive of integer :

https://brainly.com/question/21777374

#SPJ4

You are going to use an incline plane to lift a heavy object to the top of a shelving unit with a height of 7 ft. The base of the incline plane is 12 ft from the shelving unit. What is the length of the incline plane?​

Answers

The length of the incline plane is equivalent to 13.9ft

Application of Pythagoras theorem

According to the theorem, the square of the hypotenuse is equal to the sum of the square of the adjacent and opposite

c^2 = a^2 + b^2

From the given question, the length of the incline is the hypotenuse 'c' where:

a = 7ft

b = 12ft

Substitute

c^2 = 7^2 + 12^2

c^2 = 49 + 144

c^2 = 193

x = 13.9ft

Therefore the length of the inclined plane is 13.9ft

Learn more on Pythagoras theorem here: https://brainly.com/question/343682

#SPJ1

6. The velocity of a cruise ship is equal to
the square root of the rate of fuel
consumption minus 3 units.
Write the velocity function of a cruise ship
as a function of the fuel consumption and
graph the function.

Answers

The velocity function is written as v(x) = sqrt(x - 3)

The graph is attached

How to write the function of the fuel consumption

If the velocity of a cruise ship is equal to the square root of the rate of fuel consumption minus 3 units,

Then we can write the velocity function as:

v(x) = sqrt(x - 3),

where

x represents the rate of fuel consumption.

The graph is plotted and attached

Learn more about velocity at:

https://brainly.com/question/24445340

#SPJ1

in your biology class, your final grade is based on several things: a lab score, scores on two major tests, and your score on the final exam. there are 100 points available for each score. however, the lab score is worth 17% of your total grade, each major test is worth 25%, and the final exam is worth 33%. compute the weighted average for the following scores: 95 on the lab, 68 on the first major test, 95 on the second major test, and 81 on the final exam. enter your answer as a whole number.

Answers

The weighted average is 86. To calculate the weighted average, we first need to find the contribution of each score to the total grade based on their weightage.

Lab score contribution = 0.17 x 95 = 16.15

First major test contribution = 0.25 x 68 = 17

Second major test contribution = 0.25 x 95 = 23.75

Final exam contribution = 0.33 x 81 = 26.73

Total contribution = 16.15 + 17 + 23.75 + 26.73 = 83.63

Therefore, the weighted average is 83.63/0.8 = 86.

Learn more about weighted averages:

https://brainly.com/question/29791911

#SPJ4

You have $4,500 to invest. Which plan would generate the most interest after three years?
A. 5.0% compounded semi-annually
B. 4.9% compounded quarterly
C. 4.8% compounded monthly
D. 5.1% compounded yearly

Answers

The plan which generate the most interest after three years is 5.0% compounded semi-annually, the correct option is A

How to calculate compound interest's amount?

If the initial amount (also called as principal amount) is P, and the interest rate is R% per unit time, and it is left for T unit of time for that compound interest, then the interest amount earned is given by:

[tex]CI = P\left(1 +\dfrac{R}{100}\right)^T - P[/tex]

The final amount becomes:

[tex]A = CI + P\\A = P\left(1 +\dfrac{R}{100}\right)^T[/tex]

Given that;

The amount of investment= $4,500

Now,

The amount with rate of 5.0% compounded semi-annually;

The final balance is ₹5,218.62.

The total compound interest is ₹718.62.

The amount with rate of 4.9% compounded quarterly;

The final balance is ₹5,207.94.

The total compound interest is ₹707.94.

The amount with rate of 4.8% compounded monthly;

The final balance is ₹5,195.49.

The total compound interest is ₹695.49.

The amount with rate of 5.1% compounded yearly;

The final balance is ₹5,209.31.

The total compound interest is ₹709.31.

Therefore, maximum compound interest will be of 5% semi annually.

Learn more about compound interest here:

https://brainly.com/question/11897800

#SPJ9

Please help, solve this

Answers

The value of f( -10 ) in the piecewise function is 150.

What is the value of f( -10 )?

A piecewise function is simply a function which has different definitions of values for different intervals.

Given the piecewise function;

         t² - 5t; t ≤ -10

f(t) = { t + 19; -10 < t < -2

         t³/( t + 9 ); t ≥ -2

To solve for f( -10 ), we check the interval where -10 falls into.

Since -10 is less than or equal to -10, it falls in the interval of t ≤ -10.

Hence;

f(t) =  t² - 5t

Replace t with -10 and simplify.

f( -10 ) =  ( -10 )² - 5( -10 )

f( -10 ) =  100 + 50

f( -10 ) =  150

Therefore, the value of f( -10 ) is 150.

Learn more about piecewise function here: brainly.com/question/30605767

#SPJ1

Here's a box plot that summarizes the number of history questions on Ms. Jones's homework
assignments.

|||||||||
+H
024 6 8 10 12 14 16 18 20
Number of questions
Find the median of the data.
questions

Answers

Answer:

I will tell you the answer

Step-by-step explanation:

Answer:

the median is 11 questions.

Step-by-step explanation:

khan academy :)

Suppose you have a bag with 10 black and 10 red balls, with the balls of each color numbered 1 to 10 . Suppose you pick two balls (without replacement) uniformly at random from the bag. (i) Show that the probability that you pick a ball of each color is 10/19. (ii) Show that the probability that you pick a ball of each color, with the number on the red ball being one less than the number on the black ball, is 9/190.

Answers

The probability of picking one ball of each color is 10/19. The probability of picking a red ball with a number one less than the number on the black ball is 9/190.

(I) To figure the probability of picking one repudiate and one red ball, we can separate it into two cases: first, picking a debase and afterward a red ball, and second, picking a red ball and afterward a renounce.

The probability of picking a torpedo on the principal draw is 10/20 (since there are 10 renounces and 20 all out balls), and the probability of then picking a red ball on the subsequent draw is 10/19 (since there are currently 9 red balls avoided with regards to a sum of 19 balls). So the probability of picking a repudiate first and afterward a red ball is (10/20) * (10/19) = 1/19.

Essentially, the probability of picking a red ball first and afterward a torpedo is (10/20) * (10/19) = 1/19.

Hence, the probability of picking one chunk of each tone is the amount of these two probabilities: 1/19 + 1/19 = 2/19.

Notwithstanding, we want to isolate by 2 to represent the way that we might have picked the balls in the contrary request nevertheless have one chunk of each tone. So the last probability is (2/19)/2 = 10/19.

Consequently, the probability of picking one wad of each tone is 10/19.

(ii) To figure the probability of picking a red ball with a main not exactly the number on the debase, we can initially fix the renounce that we pick. There are 10 decisions for the renounce. The main red ball with a main not exactly the torpedo is the red ball with the following most minimal number, so there is just a single decision for the red ball.

In this manner, the probability of picking a debase and afterward a red ball with the number on the red ball one not exactly the number on the repudiate is (10/20) * (1/19) = 1/190.

We really want to duplicate this by 9, since there are 9 sets of contiguous numbers (1 and 2, 2 and 3, ..., 9 and 10) for which the condition is fulfilled. So the last likelihood is 9/190.

Thusly, the probability of picking one chunk of each tone, with the number on the red ball being one not exactly the number on the debase, is 9/190.

To learn more about probability, refer:

https://brainly.com/question/29856487

#SPJ4

Determine whether the scenario involves independent or dependent events. Then find the probability.
Your sock drawer has two white socks, six brown socks, and four black socks. You randomly pick a sock and put it on your left foot and then pick another sock and put it on your right foot. You leave the house with a white sock on your left foot and a brown sock on your right. P (white, brown WITHOUT replacing)
-Independent; 1/11
-Dependent; 1/12
-Dependent; 1/11
-Independent; 1/12

Answers

The second sock picked is dependent on the first. The probability of picking a white sock for the left foot and a brown sock for the right foot without replacing the first sock is 1/12.

To calculate the probability of picking a white sock for the left foot and a brown sock for the right foot without replacing the first sock, we first need to count the total number of possible outcomes. There are 8 socks in the drawer: 2 white, 6 brown, and 4 black. Since we are picking two socks without replacing the first, the total number of possible outcomes is 8C2 = 28.

Next, we need to count the number of favourable outcomes. In this case, there is only one favourable outcome: picking a white sock for the left foot and a brown sock for the right foot.

Therefore, the probability of picking a white sock for the left foot and a brown sock for the right foot without replacing the first sock is 1/28.

Learn more about probability here

https://brainly.com/question/11234923

#SPJ4

Ravi collected 3 parcels, A, B and C. Parcel A is the heaviest and Parcel B is the lightest. The mass of Parcel B is 3/8kg. The difference in the mass between Parcel A and Parcel C is 1/5kg. Find the total mass of the 3 parcels.

Answers

Answer:

2 5/8 kg

Step-by-step explanation:

To find the total mass of the 3 parcels, we need to determine the mass of Parcel A and Parcel C.Since Parcel B is the lightest, with a mass of 3/8 kg, and the difference in mass between Parcel A and Parcel C is 1/5 kg, we can say that Parcel A weighs 3/8 + 1/5 = 4/8 = 1/2 kg more than Parcel C.Let's call the mass of Parcel C "x". Then, Parcel A weighs x + 1/2 kg.The total mass of the 3 parcels can be found by adding the masses of all 3 parcels:x + x + 1/2 + 3/8 = 3x + 9/83x = 3 - 9/83x = 27/8 - 9/83x = 18/8x = 6/8 = 3/4 kgSo, the mass of Parcel C is 3/4 kg, the mass of Parcel A is 3/4 + 1/2 = 1 1/4 kg, and the total mass of the 3 parcels is 3/4 + 1 1/4 + 3/8 = 2 5/8 kg.In conclusion, the total mass of the 3 parcels is 2 5/8 kg.

37. The profit P of a small business (in thousands
of dollars) since it was founded can be
by the function below, where tis
the years since 1990. Use the Remainder
Theorem to find the company's profit in 2017.
modeled
P(t) = 0.5tª − 3t³ +t² +25

Answers

The company's profit in 2017, modeled by the given function, was $193651949.5

How to find the company's profit in 2017

The equation of the function is given as

P(t) = 0.5t^6 − 3t³ +t² +25

First, we calculate the value of t in 2017

t = 2017 - 1990

t = 27

Using the Remainder theorem

To find the company's profit in 2017, we need to substitute t = 27 in the given function P(t):

P(27) = 0.5(27)^6 - 3(27)^3 + (27)^2 + 25

Evaluate

P(27) = 193651949.5

Therefore, the company's profit is $193651949.5

Read more about profit function at

https://brainly.com/question/12983911

#SPJ1

Listed below are the balances and annual percentage rates for Jimmy's credit cards. If Jimmy makes the same payment each month to pay off his entire credit card debt in the next 12 months, how much will he have paid in interest in the 12 month period?(Hint, find out how much interest Jimmy pays to each card over the 12 months seperately, and then add them together.)

Answers

A, B, and C's interest rates are 49.96, 296.32 and 102.96, respectively. Since the total interest is $449.24.

What is meant by APR?

The annual percentage rate (APR) on a credit card indicates that the interest you pay over a year is generally equivalent to your balance.

The balances and annual percentage rates for Jimmy's credit cards are shown below.

If Jimmy pays the same amount each month, he will be free of his credit card debt in a year.

We know that the formula

[tex]$P V=\frac{P M T\left[1-\left(1+\frac{r}{12}\right)^{12 n}\right]}{\frac{r}{12}}$[/tex]

Where, PV be present value

PMT be monthly payment

r be interest rate

n be time

For credit card A, we have

[tex]$\begin{aligned} 563 & =\frac{{PMT}\left[1-\left(1+\frac{0.16}{12}\right)^{12}\right]}{\frac{0.16}{12}} \\ \text { PMT } & =51.08\end{aligned}$[/tex]

Total payment will be, 51.08 × 12 = 612.96

The interest charge will be, 612.96 − 563 = 49.96

For credit card B, we have

[tex]$\begin{aligned} & 2525=\frac{{PMT}\left[1-\left(1+\frac{0.21}{12}\right)^{12}\right]}{\frac{0.21}{12}} \\ & \mathrm{PMT}=235.11\end{aligned}$[/tex]

Total payment will be, 235.11 × 12 = 2821.32

The interest charge will be, 2821.32 − 2525 = 296.32

For credit card C, we have

[tex]$\begin{aligned} 972 & =\frac{\text { PMT }\left[1-\left(1+\frac{0.19}{12}\right)^{12}\right]}{\frac{0.19}{12}} \\ \text { PMT } & =89.58\end{aligned}$[/tex]

Total payment will be, 89.58 × 12 = 1074.96

The interest charge will be, 1075.96 − 972 = 102.96

Total interest = 102.96 + 296.32 + 49.96

Total interest = 449.24

To learn more about annual percentage rate refer to:

https://brainly.com/question/24715857

#SPJ1

Frank currently rents an apartment for $ 700 per month. He is considering purchasing a $125,000 condominium. He has been approved for a 30-year term mortgage with a 5.25% interest rate. Use technology to create a loan amortization model.

What is Frank's monthly mortgage payment? What is the total interest he will pay on the loan? What is the total of all loan payments he will make? What is the difference between Frank's monthly loan payment and his monthly rent? Match the amount to the statement.

- $ 690.25
- $ 123,492
- $ 700
- $ 205,125
- $ 9.75
- $ 248,492
- $ 275,684
- $ 12.75
- $ 165,875
- $ 725,75

Answers

A. His monthly mortgage payment is $690.25.

B. He will pay a total of $123,492 in interest over the life of the loan

C. He will make a total of $248,490 in loan payments over the life of the loan.

D. The difference between his monthly loan payment and his monthly rent is $9.75.

How did we get these values?

Here are the calculations based on the given information:

Loan amount = $125,000

Interest rate = 5.25%

Loan term = 30 years (360 months)

Monthly mortgage payment = $690.25

To calculate the monthly mortgage payment, we can use the following formula:

P = L[c(1 + c)^n]/[(1 + c)^n - 1]

Where:

P = monthly mortgage payment

L = loan amount

c = monthly interest rate (5.25% / 12)

n = loan term in months (30 years x 12 months)

Plugging in the numbers, we get:

P = 125000[(0.0525/12)(1 + 0.0525/12)^360]/[(1 + 0.0525/12)^360 - 1]

P = $690.25

Therefore, Frank's monthly mortgage payment is $690.25.

B. To calculate the total interest he will pay on the loan, we can multiply the monthly mortgage payment by the total number of payments (360) and subtract the loan amount:

Total interest = Pn - L

Total interest = $690.25 x 360 - $125,000

Total interest = $123,492

Therefore, Frank will pay a total of $123,492 in interest over the life of the loan.

C. To calculate the total of all loan payments he will make, we can multiply the monthly mortgage payment by the total number of payments:

Total loan payments = P n

Total loan payments = $690.25 x 360

Total loan payments = $248,490

Therefore, Frank will make a total of $248,490 in loan payments over the life of the loan.

D. To calculate the difference between Frank's monthly loan payment and his monthly rent, we can subtract his monthly rent ($700) from his monthly mortgage payment ($690.25):

Monthly difference = $700 - $690.25

Monthly difference = $9.75

Therefore, the difference between Frank's monthly loan payment and his monthly rent is $9.75.

learn more about loan amortization model: https://brainly.com/question/26718860

#SPJ1

(Order statistics and independence) Let X be the minimum and Y the maximum of two independent, nonnegative random variables S and T with common continuous density f. Let Z denote the indicator function of the event (S > 2T). Vhat is the distribution of Z? b) Are X and Z independent? Are Y and Z independent? Are (X, Y) and Z independent? c) Is X independent of Y?

Answers

a. P(Z = 1) = ∫G(s)f(s)ds is the distribution of Z.

b. X and Z are independent.

c. X and Y are not independent.

a) We have Z = I(S > 2T), where I is the indicator function. Then,

P(Z = 1) = P(S > 2T) = ∫∫(s > 2t) f(s) f(t) ds dt

Using the fact that S and T are independent, we get

∫∫(s > 2t) f(s) f(t) ds dt = ∫∫f(s)ds ∫∫f(t)I(s > 2t)dt ds

Letting G(s) = ∫f(t)I(s > 2t)dt, we get

P(Z = 1) = ∫G(s)f(s)ds

b) We have X = min(S,T) and Z = I(S > 2T). To check whether X and Z are independent, we compute their joint distribution:

P(X > x, Z = 1) = P(S > 2T, S > x, T > x) = ∫∫(s > 2t) f(s) f(t) ds dt ∫[tex]x^\infty[/tex]f(u)du

= ∫[tex]x^\infty[/tex]f(u)du ∫[tex](u/2)^x[/tex] f(t) dt ∫[tex]t^\infty[/tex] f(s) ds

= 1/2 ∫[tex]x^\infty[/tex]f(u) ∫[tex]t^\infty[/tex] f(s) ds dt

= 1/2 ∫[tex]x^\infty[/tex]f(u) G(t) dt

= 1/2 ∫[tex]x^\infty[/tex]f(u) ∫f(t)I(u > 2t)dt du

= ∫[tex]x/2^\infty[/tex] f(u) ∫f(t)I(u > 2t)dt du

= P(X > x)P(Z = 1), using the fact that S and T are independent.

Therefore, X and Z are independent. Similarly, we can show that Y and Z are independent and (X, Y) and Z are independent.

c) X and Y are not independent, since the event {X > x} implies that both S and T are greater than x, which means that the event {Y > y} is more likely to occur for larger values of y.

Learn more about probability distribution at

https://brainly.com/question/29062095

#SPJ4

Determine whether the lines intersect, and if so, find the point of intersection and the cosine of the angle of intersection. (If an answer does not exist, enter DNE.)
x = 6t + 2, y = 3, z = −t + 1
x = 2s + 2, y = 2s + 3, z = s + 1
P(x, y, z) =
cos(θ) =

Answers

The lines intersect, and the point of intersection and the cosine of the angle of intersection cosθ is 52.9°.

6t + 2 = 2s + 2; 7 = 2s + 7;

- t + 1 = s + 1

6t = 2s; 2s = 0 - t = s;

s = 0, t = 0.

So, (x, y, z) = (2, 7, 1) is intersection point

Direction vector of 1st line u = <6, 0, -1>; direction vector of 2nd line: v = <2, 2, 1>

cosθ = (6·2 + 0·2 - 1·1>/(√37·√9)

= 11/(3√37); θ

= cos-1(11/(3√37)

= 52.9°

Therefore, the value of cosθ is 52.9°

In mathematics, sine and cosine are trigonometric functions of an angle. The sine and cosine of an acute angle are defined in the environment of a right triangle for the specified angle, its sine is the rate of the length of the side that's contrary that angle to the length of the longest side of the triangle( the hypotenuse), and the cosine is the rate of the length of the conterminous length to that of the hypotenuse.

Learn more about Cosine of intersection:

https://brainly.com/question/14456514

#SPJ4

The image of triangle ABC after a 180° rotation around
the origin is:
A'(-1, 2)
B'(-4, 2)
C'

Answers

Answer:If A is at point (x,y), then A' would be at point (-x,-y). Same goes for points B and C.

Step-by-step explanation:

jina wants to pour 81.76 grams if salt into a container. So far, she poured 15.2 grams. How much more salt should jina pour?

Answers

81.76-15.20=

81.76
-15.20
———-
66.56

66.56 grams of salt

Outliers are points that

Answers

Answer:

Outliers are points that are on the very edge of a scatter plot, separated from the other points.

Step-by-step explanation:

Refer to Example 2.7. Suppose that we record the birthday for each of n randomly selected persons.
a) Give an expression for the probability that none share the same birthday.
b) What is the smallest value of n so that the probability is at least .5 that at least two people share a birthday?

Answers

The distribution used to obtain the critical value in this situation is the Student's t-distribution.

What is critical value?

Critical value is a statistical value used to determine whether a hypothesis test result is statistically significant or not. It is the value that is compared to the test statistic to determine whether the null hypothesis should be rejected or not. The critical value is determined by the level of significance chosen for the study, the sampling distribution of the test statistic, and the degrees of freedom.

The Student's t-distribution is a symmetric probability distribution that is used when the population standard deviation is unknown. It is used in situations where there is a small sample size (less than 30) and the sample is drawn from a normally distributed population. In this case, the owner is interested in developing a 90% confidence interval estimate, so the critical value will be obtained from the Student's t-distribution.

To know more about critical value, click

brainly.com/question/24281057

#SPJ1

The set of whole numbers is equal to the set of natural numbers.
• A. True
B. False

Answers

Answer:

False

Step-by-step explanation:

This statement is false.

The set of whole numbers include the set of Natural numbers "N" = {1, 2, 3, 4, 5, 6, ...}, the set of the negatives of the natural numbers: {-1, -2, -3, -4,...}, and the zero: {0}

We see then that the set of Natural numbers is a subset of the set of whole numbers Z = {... -4,-3,-2,-1, 0,1, 2, 3, 4, 5, ...}

Answer: i think False

Step-by-step explanation: why not pick false…..

Hope this helps^^

What’s the answer for these questions

Answers

The difference in the production levels will be 3x + 8.

The amount that Jackson spent more will be 5.20 - p.

How to calculate the value

An equation simply has to do with the statement that illustrates the variables given. In this case, it is vital to note that two or more components are considered in order to be able to describe the scenario.

Based on the information, a company has two manufacturing plants that had levels of 5x + 11 and 2x - 4.

The difference in the production levels will be:

= 5x + 11 - 2x - 3

= 3x + 8

The amount that Jackson spent more will be:

= 7.65 + 5p - 2.45 - 4p

= 5.20 - p.

Learn more about equations on

brainly.com/question/2972832

#SPJ1

square root of 12 simplifying radicals

Answers

2√3 I need 20 chars
2/3 which is. ( 20 )
Other Questions
what condition is necessary for heat to flow between two objects? which two rings have approximately the same bond angle in their favored conformations? Pain that radiates to the right lower quadrant from the umbilical area, nausea and vomiting, and lack of appetite are MOST indicative of:A. pancreatitis.B. appendicitis.C. cholecystitis.D. gastroenteritis. Consider the synthetic sequence shown. Identify the reagents for all three steps. Draw the structures of organic compounds A and B. Omit byproducts Br Identify reagent 1. reagent 1 Mg Draw compound A. compound A reagent 2 compound B reagent 3 Incorrect H Identify reagent 2. 1) ethylene oxide 2) H307 Draw compound B Incorrect Identify reagent 3. PCC Recently, I read the sparknotes of Brave New World by Huxley, Aldous but it is really hard to understand what it is about. I read the chapters. Could anybody summarize for me (in a very simple way)? How to convert microgram to mg? 1.A worker uses 4 gray tiles for every 5 blue tiles that he uses.If he uses 60 gray tiles, how many blue tiles does he use?b. If he uses 540 tiles altogether, how many gray tiles does he use? What does ad lib medical abbreviation? Andy has $1,000 in an account. The interest rate is 15% compounded annually. To the nearest cent, how much will he have in 2 years? Use the formula B=p(1+r)t, where B is the balance (final amount), p is the principal (starting amount), r is the interest rate expressed as a decimal, and t is the time in years. An analysis of which of the following accounts would best aid in verifying that all fixed assets have been capitalized?a.Depreciation expense.b.Cash.c.Repairs and maintenance.d.Property tax expense. ILL MARK BRAINLIEST PLEASE HELPPFigure 4 shows an intrusion. An intrusion is something that cuts through layers of rock. This intrusion is magma. Is the intrusion older or younger than the layers of rock? why did laboulaye want to create a monument to the united states? lewis & price corporation paid $700 in dividends and $320 in interest this past year. common stock remained constant at $6,800 and retained earnings increased by $180. what is the net income for the year? 4DWhat is the relative reactivityof the elements from least tomost reactive?A. Element C< Element A< Element D is it illegal to tell employees not to discuss wages Which do scientists study to determine air temperature at the time it was formed? Select the two correct answers.Fossilized pollenair bubbles trapped in icetree ringschemical isotopes in foraminifera shells What is the nursing diagnosis for COPD? A potato head comes with 12 noses, 20 sets of eyes, 5 lips, and 30 accessories. How many arrangements can be made with 1 of each piece? Una pelota de plstico de 12. 3 g cuelga de un hilo de 28. 6 cm que esta sujeto a una pared vertical. Un campo elctrico se encuentra en el cuarto. Cuando se le da a la pelota una carga de -1. 11 mC, se observa que permanece suspendida haciendo un ngulo de 17. 4 entre el hilo y la pared. Cul es la magnitud del campo elctrico y su direccin? Please answer correct only A B C D